Une fonction et sa transformée de Fourier — Les-mathematiques.net The most powerful custom community solution in the world

Une fonction et sa transformée de Fourier

Bonjour. Je serais très reconnaissant si quelqu'un pouvait m'aider à faire ceci.
J'ai le résultat suivant.

Si $f\in L^1(\Bbb R)$ et $\int_{\Bbb R}\int_{\Bbb R} |f(x)||\hat f(y)| e^{|x||y|} dxdy<\infty$, alors $f=0$
(avec $\hat{f}$ et la transformée de Fourier de $f$).

D'après le résultat cité en haut je veux montrer ceci.
Supposons que $ab>\frac{1}{4}$. Si $f(x)=O(e^{-ax^2})$ et $\hat{f}(y)=O(e^{-by^2})$, alors $f=0$.
Comment je puis faire et merci.

Notez que $\hat{f}(e^{-ax^2})(y)=e^{-\frac{1}{4a}y^2}$.

Réponses

  • La stratégie semble pourtant assez claire : montre que $\int_{\Bbb R}\int_{\Bbb R} |f(x)||\hat f(y)| e^{|x||y|} dxdy<\infty$ en utilisant les estimations données sur $f$ et $\hat f$, c'est assez immédiat.
  • D'après les hypothèses j'ai $|f(x)||\hat f(y)| e^{|x||y|}<C e^{-ax^2-\frac{1}{4a}y^2+|x||y|}$

    Mais $\int_{\Bbb R}\int_{\Bbb R}e^{-ax^2-\frac{1}{4a}y^2+|x||y|}=+\infty$.
    Si $\int_{\Bbb R}\int_{\Bbb R}e^{-ax^2-\frac{1}{4a}y^2+|x||y|}<+\infty$ alors le résultat est immédiat.
  • Oui je dois décomposer l'intégrale et utiliser les hypothèses, merci @Poirot
  • Où as-tu vu que ton intégrale double valait l'infini ? Si $a > 0$ est fixé, l'intégrale est (largement) convergente.
  • Merci@noix de totos, voir mon dernier post
  • Obscur.
  • Pourquoi cher @ P.
  • Bonjour,
    mathspe a écrit:
    remarquer que $\hat{f}(e^{-ax^2})(y)=e^{-\frac{1}{4a}y^2}$ et j'ai utilisé le résultat cité en haut

    Non, c'est ${\cal F}(e^{-ax^2})(y)=e^{-\frac{1}{4a}y^2}$ où $\cal F$ désigne l'opérateur transformée de Fourier. $\hat f$ ça désigne une fonction, pas l'opérateur transformée de Fourier. C'est $\hat f = {\cal F}(f)$.

    Et une pensée horrible m'a traversée. Est-ce que tu as dit que $f(x)=O(e^{-ax^2})\Rightarrow \hat f(y) = O({\cal F}(e^{-ax^2})(y))= O( e^{-\frac{1}{4a}y^2})$ ? Parce que cette implication est fausse.
  • Bonjour @Calli,
    juste une notation je designe par $\hat{f}$ la transformée de Fourier de $f$, que tu as notée par ${\cal F}(e^{-ax^2})(y)=e^{-\frac{1}{4a}y^2}$.

    J'ai le résultat suivant.
    Si $f\in L^1(\Bbb R)$ et $\int_{\Bbb R}\int_{\Bbb R} |f(x)||\hat f(y)| e^ {|x||y|} dxdy<\infty$, alors $f=0$,
    et je vais l'utiliser pour montrer que
    $\int_{\Bbb R}\int_{\Bbb R}e^{-ax^2-\frac{1}{4a}y^2+|x||y|}=+\infty$.

    En effet, je note par $f(x)=e^{-ax^2}$ donc $\hat{f}(y)=e^{-\frac{1}{4a}y^2}$ donc
    $\int_{\Bbb R}\int_{\Bbb R}e^{-ax^2-\frac{1}{4a}y^2+|x||y|}=\int_{\Bbb R}\int_{\Bbb R} |f(x)||\hat f(y)| e^ {|x||y|} dxdy=\infty$

    Pour ta question, oui l'implication est fausse. En fait, j'aurais du écrire ceci.
    Comme hypothèse
    $f(x)=O(e^{-ax^2})$ et $\hat f(y) = O(e^{-by^2})$, or $ ab>\frac{1}{4}$ alors $-by^2<-\frac{1}{4a}y^2$ par conséquent,
    $\hat f(y) = O(e^{-\frac{1}{4a}y^2})$.
    Merci.
  • Aucune idée
  • On peut montrer plus simplement ce que tu as montré en utilisant Beurling, à savoir :
    $$\int_{\mathbb R^2} e^{-ax^2- \frac{1}{4a}y^2+|x||y|} \mathrm dx \mathrm dy=\infty.
    $$ Il suffit juste d'écrire :
    $$-ax^2 - \frac{1}{4a}y^2-|x||y| = -a \left (|x|+ \frac{1}{2a}|y| \right ) ^2 ,
    $$ et de faire les bons changements de variables.
    Mais par contre, si $ab > \frac 1 4$ (strictement, donc), là ça marche (pour la même raison).
  • Merci pour ton aide, Pour le cas $ab>1/4$, je n'arrive pas à comprendre ta méthode
  • Écrire $b=\frac{1}{4a} + \varepsilon$

    D'ailleurs, es-tu sûr que la transformée de Fourier de $x \mapsto e^{-ax^2}$ est bien $y \mapsto e^{-\frac{1}{4a}y^2}$ ?
  • Oui la transformée de Fourier est correcte.
  • Hum.
Connectez-vous ou Inscrivez-vous pour répondre.
Success message!